figshare
Browse
manuscript.pdf (110.34 kB)

The Riemann Hypothesis

Download (110.34 kB)
Version 20 2021-07-14, 21:06
Version 19 2021-07-08, 20:07
Version 18 2021-07-08, 15:16
Version 17 2021-07-08, 06:09
Version 16 2021-07-08, 06:08
Version 15 2021-07-07, 20:19
Version 14 2021-07-01, 03:28
Version 13 2021-07-01, 03:28
Version 12 2021-05-11, 05:28
Version 11 2021-05-11, 04:06
Version 10 2021-05-11, 00:45
Version 9 2021-05-08, 08:10
Version 8 2021-05-07, 08:07
Version 7 2021-04-26, 06:10
Version 6 2021-03-30, 05:59
Version 5 2021-03-24, 04:43
Version 4 2021-03-23, 23:41
Version 3 2021-03-20, 04:15
Version 2 2021-03-19, 03:06
Version 1 2018-05-28, 11:17
preprint
posted on 2021-07-14, 21:06 authored by Frank VegaFrank Vega
For every prime number $p_{n}$, we define the sequence $X_{n} = \frac{\prod_{q \mid N_{n}} \frac{q}{q-1}}{e^{\gamma} \times \log\log N_{n}}$, where $N_{n} = \prod_{k = 1}^{n} p_{k}$ is the primorial number of order $n$ and $\gamma \approx 0.57721$ is the Euler-Mascheroni constant. The Nicolas theorem states that the Riemann hypothesis is true if and only if the $X_{n} > 1$ holds for all prime $p_{n} > 2$. For every prime number $p_{k}$, $X_{k} > 1$ is called the Nicolas inequality. We show if the sequence $X_{n}$ is strictly decreasing for $n$ big enough, then the Riemann hypothesis should be true. Moreover, we demonstrate that the sequence $X_{n}$ is indeed strictly decreasing when $n \to \infty$. Notice that, Choie, Planat and Sol{\'e} in the preprint paper arXiv:1012.3613 have a proof that the Cram{\'e}r conjecture is false when $X_{n}$ is strictly decreasing for $n$ big enough. This paper is an extension of their result.

History

Usage metrics

    Licence

    Exports

    RefWorks
    BibTeX
    Ref. manager
    Endnote
    DataCite
    NLM
    DC